Sr Examen

Otras calculadoras:

Límite de la función 1/n+n^(-2)+n^3

cuando
v

Para puntos concretos:

Gráfico:

interior superior

Definida a trozos:

Solución

Ha introducido [src]
     /1   1     3\
 lim |- + -- + n |
n->oo|n    2     |
     \    n      /
$$\lim_{n \to \infty}\left(n^{3} + \left(\frac{1}{n} + \frac{1}{n^{2}}\right)\right)$$
Limit(1/n + n^(-2) + n^3, n, oo, dir='-')
Método de l'Hopital
Tenemos la indeterminación de tipo
oo/oo,

tal que el límite para el numerador es
$$\lim_{n \to \infty}\left(n^{5} + n + 1\right) = \infty$$
y el límite para el denominador es
$$\lim_{n \to \infty} n^{2} = \infty$$
Vamos a probar las derivadas del numerador y denominador hasta eliminar la indeterminación.
$$\lim_{n \to \infty}\left(n^{3} + \left(\frac{1}{n} + \frac{1}{n^{2}}\right)\right)$$
=
Introducimos una pequeña modificación de la función bajo el signo del límite
$$\lim_{n \to \infty}\left(\frac{n^{5} + n + 1}{n^{2}}\right)$$
=
$$\lim_{n \to \infty}\left(\frac{\frac{d}{d n} \left(n^{5} + n + 1\right)}{\frac{d}{d n} n^{2}}\right)$$
=
$$\lim_{n \to \infty}\left(\frac{5 n^{4} + 1}{2 n}\right)$$
=
$$\lim_{n \to \infty}\left(\frac{\frac{d}{d n} \left(5 n^{4} + 1\right)}{\frac{d}{d n} 2 n}\right)$$
=
$$\lim_{n \to \infty}\left(10 n^{3}\right)$$
=
$$\lim_{n \to \infty}\left(10 n^{3}\right)$$
=
$$\infty$$
Como puedes ver, hemos aplicado el método de l'Hopital (utilizando la derivada del numerador y denominador) 2 vez (veces)
Gráfica
Respuesta rápida [src]
oo
$$\infty$$
Otros límites con n→0, -oo, +oo, 1
$$\lim_{n \to \infty}\left(n^{3} + \left(\frac{1}{n} + \frac{1}{n^{2}}\right)\right) = \infty$$
$$\lim_{n \to 0^-}\left(n^{3} + \left(\frac{1}{n} + \frac{1}{n^{2}}\right)\right) = \infty$$
Más detalles con n→0 a la izquierda
$$\lim_{n \to 0^+}\left(n^{3} + \left(\frac{1}{n} + \frac{1}{n^{2}}\right)\right) = \infty$$
Más detalles con n→0 a la derecha
$$\lim_{n \to 1^-}\left(n^{3} + \left(\frac{1}{n} + \frac{1}{n^{2}}\right)\right) = 3$$
Más detalles con n→1 a la izquierda
$$\lim_{n \to 1^+}\left(n^{3} + \left(\frac{1}{n} + \frac{1}{n^{2}}\right)\right) = 3$$
Más detalles con n→1 a la derecha
$$\lim_{n \to -\infty}\left(n^{3} + \left(\frac{1}{n} + \frac{1}{n^{2}}\right)\right) = -\infty$$
Más detalles con n→-oo